Find Solutions to a+b+c+d=4, a^2+b^2+c^2+d^2=6, a^3+b^3+c^3+d^3=94/9 in [0,2]

  • MHB
  • Thread starter anemone
  • Start date
In summary, to find solutions to the given equations, you can use algebraic manipulation and substitution. The given interval is [0,2], which means that the variables a, b, c, and d can take on values between 0 and 2, inclusive. There are several strategies that can be used to solve systems of equations, such as substitution, elimination, and graphing, with substitution being the most efficient in this case. To check if your solutions are correct, you can substitute them into the original equations or use a graphing calculator. This problem can also be solved using a computer program or calculator, but understanding the manual steps and strategies is important for effective use of the technology.
  • #1
anemone
Gold Member
MHB
POTW Director
3,883
115
Find all the solutions to the system

$a+b+c+d=4\\a^2+b^2+c^2+d^2=6\\a^3+b^3+c^3+d^3=\dfrac{94}{9}$
in $[0, 2]$.
 
Mathematics news on Phys.org
  • #2
anemone said:
Find all the solutions to the system $p_1 = a+b+c+d=4\\p_2 = a^2+b^2+c^2+d^2=6\\ p_3 = a^3+b^3+c^3+d^3=\dfrac{94}{9}$ in $[0, 2]$.
Let $x^4 - e_1x^3 + e_2x^2 - e_3x + e_4 = 0$ be the equation with roots $a,b,c,d$. By Newton's identities, $$\textstyle e_1 = p_1 = 4,\qquad e_2 = \frac12(p_1^2 - p_2) = \frac12(16 - 6) = 5,\qquad e_3 = \frac16(p_1^3 - 3p_1p_2 + 2p_3) = \frac16(64 - 72 + \frac{188}9) = \frac{58}{27}.$$ So the equation is $x^4 - 4x^3 + 5x^2 - \frac{58}{27}x + e_4 = 0$. Since $58$ is close to twice $27$, write the equation as $$x^4 - 4x^3 + 5x^2 - 2x = \tfrac4{27}x - e_4, \\ x(x-2)(x^2 - 2x + 1) = \tfrac4{27}(x-s),$$ where $s$ is a constant. Now look at the graph:

[DESMOS]{"version":7,"graph":{"viewport":{"xmin":-0.47946479454949004,"ymin":-1.7930541038513184,"xmax":2.51919243201301,"ymax":1.9552674293518066}},"randomSeed":"5a5979bd9ad99a52fd1cfa25e8de4160","expressions":{"list":[{"type":"expression","id":"1","color":"#c74440","latex":"x^{4}-4x^{3}+5x^{2}-2x"},{"type":"expression","id":"2","color":"#2d70b3","latex":"\\frac{4}{27}\\left(x-s\\right)"},{"type":"expression","id":"4","color":"#6042a6","latex":"s=2","hidden":true,"slider":{"hardMin":true,"hardMax":true,"min":"1","max":"3","step":"0.1"}},{"type":"expression","id":"5","color":"#000000"}]}}[/DESMOS]
The roots of the equation are the points where the blue line meets the red curve. By using the slider, you can see that if $s<2$ then the largest root is greater than $2$. But if $s>2$ then the blue line goes lower, and only meets the red curve in two points, which means that two of the roots of the quartic equation are complex. So for the equation to have four real roots in the interval $[0,2]$, $s$ must be equal to $2$. After multiplying by $27$ the equation then becomes $(x-2)(27x^3 - 54x^2 + 27x - 4) = 0$, which factorises as $(3x-1)^2(3x-4)(x-2) = 0$. Therefore the solutions to the system are $\{a,b,c,d\} = \{\frac13,\frac13,\frac43,2\}$ (in any order).
 
  • #3
Awesome, Opalg!(Cool) And thanks for participating!

I will start from the quartic equation $p(x)=x^4-4x^3+5x^2-\dfrac{58}{27}x+k$ where $p(x)$ has roots $a, b, c, d$.

$p'(x)=4x^3-12x^2+10x-\dfrac{58}{27}=\dfrac{2}{27}(3x-1)(18x^2-48x+29)$

Solving $p/(x)=0$ gives $x=\dfrac{1}{3},\,\dfrac{4}{3}\pm\dfrac{\sqrt{6}}{2}$.

Since $p(x)$ is a 4th degree polynomial with positive leading coefficient and $p'(x)$ has 3 distinct real roots in $(0, 2)$, it follows that in order for $a, b, c, d$ to be solutions of the given equations where $0\le a, b, c, d\le 2$, we must have

$p(0)\ge 0,\,p\left(\dfrac{1}{3}\right)\le0,\,p\left(\dfrac{4}{3}-\dfrac{\sqrt{6}}{2}\right)\ge0,\, p\left(\dfrac{4}{3}+\dfrac{\sqrt{6}}{2}\right)\le0,\,p(2)\ge 0$

Evaluating, we find $p\left(\dfrac{1}{3}\right)=p(2)=k-\dfrac{8}{27}$. Hence, $k=\dfrac{8}{27}$, from which we obtain

$\begin{align*}p(x)&=x^4-4x^3+5x^2-\dfrac{58}{27}x+\dfrac{8}{27}\\&=\dfrac{1}{27}(27x^4-108x^3+135x^2-58x+8)\\&=\dfrac{1}{27}(3x-1)^2(3x-4)(x-2)\end{align*}$

Therefore, the solutions in $[0, 2]$ are the 12 permutations of $\left(\dfrac{1}{3},\, \dfrac{1}{3},\, \dfrac{4}{3},\,2 \right)$.
 

1. What is the solution to the given equations?

The solution to the given equations is a=1, b=1, c=1, d=1.

2. How did you arrive at the solution?

The solution was found by solving the first two equations simultaneously to eliminate two variables, and then substituting the resulting values into the third equation to solve for the remaining two variables.

3. Is there more than one solution to the equations?

No, there is only one solution to the given equations.

4. Can the solution be verified?

Yes, the solution can be verified by substituting the values into the equations and checking if they satisfy the given conditions.

5. Are there any restrictions on the values of a, b, c, and d?

Yes, the values of a, b, c, and d must be between 0 and 2, inclusive, as specified in the given range.

Similar threads

  • General Math
Replies
1
Views
553
  • General Math
Replies
1
Views
765
Replies
3
Views
802
  • General Math
Replies
1
Views
837
  • General Math
Replies
1
Views
729
Replies
6
Views
961
  • General Math
Replies
2
Views
749
Replies
1
Views
932
Replies
3
Views
673
Back
Top